Difference between revisions of "2018 AMC 12B Problems/Problem 17"

(Solution 2 (requires justification))
(Solution 3)
Line 10: Line 10:
  
 
<cmath>\frac{1}{bd}=\frac{c}{d}-\frac{a}{b}=\left(\frac{c}{d}-\frac{p}{q}\right)+\left(\frac{p}{q}-\frac{a}{b}\right) \geq \frac{1}{dq}+\frac{1}{bq},</cmath>
 
<cmath>\frac{1}{bd}=\frac{c}{d}-\frac{a}{b}=\left(\frac{c}{d}-\frac{p}{q}\right)+\left(\frac{p}{q}-\frac{a}{b}\right) \geq \frac{1}{dq}+\frac{1}{bq},</cmath>
which reduces to <math>q\geq b+d</math>. We can easily find that <math>p=a+c</math>, giving an answer of <math>16-9=\boxed{7}</math>. (pieater314159)
+
which reduces to <math>q\geq b+d</math>. We can easily find that <math>p=a+c</math>, giving an answer of <math>\boxed{\textbf{(A)}\ 7}</math>. (pieater314159)
  
 
==Solution 2 (requires justification)==
 
==Solution 2 (requires justification)==
Line 22: Line 22:
 
<math>4q - 7p = 1</math>
 
<math>4q - 7p = 1</math>
  
Solving the system of equations yields <math>q=16</math> and <math>p=9</math>. Therefore, the answer is <math>16-9=\boxed{7}</math>
+
Solving the system of equations yields <math>q=16</math> and <math>p=9</math>. Therefore, the answer is <math>\boxed{\textbf{(A)}\ 7}</math>
 +
 
 +
==Solution 3==
 +
Cross-multiply the inequality to get <cmath>35q < 63p < 36q.</cmath>
 +
 
 +
Then,
 +
<cmath>0 < 63p-35q < q,</cmath>
 +
<cmath>0 < 7(9p-5q) < q.</cmath>
 +
 
 +
Since <math>p</math>, <math>q</math> are integers, <math>9p-5q</math> is an integer. To minimize <math>q</math>, start from <math>9p-5q=1</math>, which gives <math>p=\frac{5q+1}{9}</math>. This limits <math>q</math> to be greater than <math>7</math>, so test values of <math>q</math> starting from <math>q=8</math>. However, <math>q=8</math> to <math>q=14</math> do not give integer values of <math>p</math>.
 +
 
 +
Once <math>q>14</math>, it is possible for <math>9p-5q</math> to be equal to <math>2</math>, so <math>p</math> could also be equal to <math>\frac{5q+2}{9}.</math> The next value, <math>q=15</math>, is not a solution, but <math>q=16</math> gives <math>p=\frac{5\cdot 16 + 1}{9} = 9</math>. Thus, the smallest possible value of <math>q</math> is <math>16</math>, and the answer is <math>16-9= \boxed{\textbf{(A)}\ 7}</math>.
  
 
==See Also==
 
==See Also==

Revision as of 18:28, 16 February 2018

Problem

Let $p$ and $q$ be positive integers such that \[\frac{5}{9} < \frac{p}{q} < \frac{4}{7}\]and $q$ isi as small as possible. What is $q-p$?

$\textbf{(A) } 7 \qquad \textbf{(B) } 11 \qquad \textbf{(C) } 13 \qquad \textbf{(D) } 17 \qquad \textbf{(E) } 19$

Solution 1

We claim that, between any two fractions $a/b$ and $c/d$, if $bc-ad=1$, the fraction with smallest denominator between them is $\frac{a+c}{b+d}$. To prove this, we see that

\[\frac{1}{bd}=\frac{c}{d}-\frac{a}{b}=\left(\frac{c}{d}-\frac{p}{q}\right)+\left(\frac{p}{q}-\frac{a}{b}\right) \geq \frac{1}{dq}+\frac{1}{bq},\] which reduces to $q\geq b+d$. We can easily find that $p=a+c$, giving an answer of $\boxed{\textbf{(A)}\ 7}$. (pieater314159)

Solution 2 (requires justification)

Assume that the difference $\frac{p}{q} - \frac{5}{9}$ results in a fraction of the form $\frac{1}{9q}$. Then,

$9p - 5q = 1$

Also assume that the difference $\frac{4}{7} - \frac{p}{q}$ results in a fraction of the form $\frac{1}{7q}$. Then,

$4q - 7p = 1$

Solving the system of equations yields $q=16$ and $p=9$. Therefore, the answer is $\boxed{\textbf{(A)}\ 7}$

Solution 3

Cross-multiply the inequality to get \[35q < 63p < 36q.\]

Then, \[0 < 63p-35q < q,\] \[0 < 7(9p-5q) < q.\]

Since $p$, $q$ are integers, $9p-5q$ is an integer. To minimize $q$, start from $9p-5q=1$, which gives $p=\frac{5q+1}{9}$. This limits $q$ to be greater than $7$, so test values of $q$ starting from $q=8$. However, $q=8$ to $q=14$ do not give integer values of $p$.

Once $q>14$, it is possible for $9p-5q$ to be equal to $2$, so $p$ could also be equal to $\frac{5q+2}{9}.$ The next value, $q=15$, is not a solution, but $q=16$ gives $p=\frac{5\cdot 16 + 1}{9} = 9$. Thus, the smallest possible value of $q$ is $16$, and the answer is $16-9= \boxed{\textbf{(A)}\ 7}$.

See Also

2018 AMC 12B (ProblemsAnswer KeyResources)
Preceded by
Problem 16
Followed by
Problem 18
1 2 3 4 5 6 7 8 9 10 11 12 13 14 15 16 17 18 19 20 21 22 23 24 25
All AMC 12 Problems and Solutions

The problems on this page are copyrighted by the Mathematical Association of America's American Mathematics Competitions. AMC logo.png